You are on page 1of 8

ACTS 4302

Instructor: Natalia A. Humphreys


SOLUTION TO HOMEWORK 5
Lesson 8: Fitting stock prices to a lognormal distribution.
Lesson 9: The Black-Scholes formula.
Lesson 10: The Black-Scholes formula: Greeks.
Problem 1
You are given the following monthly stock prices:

St

1 38
2 42
3 59
4 45
5 37
6 45
Estimate the continuously compounded expected rate of return on the stock.
Solution. We calculate xi = ln(St /St1 ) and corresponding x2i :
xi

x2i

2 42

ln(42/38) = 0.1001

0.01

3 59

ln(59/42) = 0.3399

0.1155

St

1 38

4 45 ln(45/59) = 0.2709 0.0734


5 37 ln(37/45) = 0.1957 0.0383
6 45

ln(45/37) = 0.1957

0.0383

We need to estimate

=
+ 0.5
2 , where

= Nx

s
P 2


xi
n
2
x

= N
n1
n
N is the number of periods per year, n is the number one less than the number of observations of stock
price.
Since there are 6 observations, n = 5. Calculating
:
1 45
x
= ln
= 0.0338
5 38

=N x
= 12 0.0338 = 0.4057

Copyright Natalia A. Humphreys, 2014

ACTS 4302. AU 2014. SOLUTION TO HOMEWORK 5.

Calculating
2:
X

x2i = 0.2755


5 1
2
2

= 12
0.2755 0.0338 = 0.8094
4 5
Therefore,

= 0.4057 + 0.5 0.8094 = 0.8104

Problem 2
You are given the following statistics for weekly closing prices of a stock St :
20
X


St
= 0.03489
St1
t=1
2
20  
X
St
(ii)
ln
= 0.1796
St1

(i)

ln

t=1

Estimate the continuously compounded expected rate of return on the stock.


(A) 0.01

(B) 0.09

(C) 0.34

(D) 0.49

(E) 0.58

Solution. By definition, if xi are observed stock prices adjusted to remove the effect of dividends, the
estimate for the continuously compounded annual return is:

=
+ 0.5
2 , where
= Nx
and
s
P 2


xi
n
2
x

= N
n1
n
N is the number of periods per year, n is the number one less than the number of observations of stock
price, xi = ln (Si /Si1 ).
The sample mean is m
= 0.03489/20 = 0.001745. The sample variance is
20
s =
19
2

0.1796
0.0017452
20


= 0.00945

The estimated annual return is


= 52 (0.001745 + 0.5(0.00945)) = 0.3364

.

Problem 3
For a 9-month European put option on a stock, you are given:
(i) The stocks price is 50.
(ii) The strike price is 45.
(iii) The continuous dividend rate for the stock is 2%.
(iv) The stocks annual volatility is 15%
(v) The continuously compounded risk-free interest rate is 3%.
Determine the Black-Scholes premium for the option.
Page 2 of 8

ACTS 4302. AU 2014. SOLUTION TO HOMEWORK 5.

Copyright Natalia A. Humphreys, 2014

Solution. We are given: Eur. put, t = 0.75, S = 50, K = 45, = 0.02, = 0.15, r = 0.03.

P = Kert N (d2 ) Set N (d1 ), where



ln 50
+ 0.03 0.02 + 0.5 0.152 0.75
ln (S/K) + (r + 12 2 )t
0.1213
45

=
d1 =
=
= 0.9338 0.93
0.1299
t
0.15 0.75
N (d1 ) = N (0.9338) 1 N (0.93) = 1 0.8238 = 0.1762

d2 = d1 t = 0.9338 0.1299 = 0.8039 0.8


N (d2 ) = N (0.8039) 1 N (0.8) = 1 0.7881 = 0.2119
Kert = 45e0.030.75 = 43.9988
Set = 50e0.020.75 = 49.2556
P = 43.9988 0.2119 49.2556 0.1762 = 9.3233 8.6788 = 0.6445

Problem 4
For a 9-month European call option on a stock, you are given:
(i) The stocks price is 60.
(ii) The strike price is 70.
(iii) = 0.4.
(iv) The continuously compounded risk-free interest rate is 5%.
(v) The stock pays no dividend.
Determine the change in the Black-Scholes premium for the option if the stock pays a quarterly dividend
of 1 with the first dividend payable 3 months after the option is written, and the expiry occurs after
the 3rd dividend.
Solution. We are given: Eur. call, t = 0.75, S = 60, K = 70, = 0.4, r = 0.05.

If = 0, then

C = Set N (d1 ) Kert N (d2 ), where



ln 60
+ 0.05 0 + 0.5 0.42 0.75
ln (S/K) + (r + 12 2 )t
0.0567
70

d1 =
=
=
= 0.1635 0.16
0.3464
t
0.4 0.75
N (d1 ) = N (0.1635) 1 N (0.16) = 1 0.5636 = 0.4364

d2 = d1 t = 0.1635 0.3464 = 0.5099 0.51


N (d2 ) = N (0.5099) 1 N (0.51) = 1 0.695 = 0.305
Kert = 70e0.050.75 = 67.4236
Set = 60
C = 60 0.4364 67.4236 0.305 = 26.184 20.5642 = 5.6198

If the dividend of 1 is paid quarterly, then the pre-paid forward price of the stock is:

F P (S) = 60 1 e0.050.25 1 e0.050.5 1 e0.050.75 = 60 2.9261 = 57.0739


Page 3 of 8

ACTS 4302. AU 2014. SOLUTION TO HOMEWORK 5.

Copyright Natalia A. Humphreys, 2014

Using F P (S) for S and = 0 in the formula for C above, we obtain:


C = Set N (d1 ) Kert N (d2 ), where

ln 57.0739
+ 0.05 0 + 0.5 0.42 0.75
ln (S/K) + (r + 12 2 )t
0.1066
70

d1 =
=
=
= 0.3079 0.31
0.3464
t
0.4 0.75
N (d1 ) = N (0.3079) 1 N (0.31) = 1 0.6217 = 0.3783

d2 = d1 t = 0.3079 0.3464 = 0.6543 0.65


N (d2 ) = N (0.6543) 1 N (0.65) = 1 0.7422 = 0.2578
Kert = 70e0.050.75 = 67.4236
Set = 57.0739
C = 57.0739 0.3783 67.4236 0.2578 = 21.5911 17.3818 = 4.2093
Thus, the difference between these two options is
= 5.6198 4.2093 = 1.4105

Problem 5
For a yen-dollar exchange rate, you are given:
(i) The spot exchange rate is 120U/$.
(ii) The annual volatility of the exchange rate is 20%.
(iii) The continuously compounded risk-free rate for dollars is 0.06.
(iv) The continuously compounded risk-free rate for yen is 0.02.
Determine the Garman-Kohlhagen premium for a yen-denominated 6-month European put option on
dollars with a strike price of 115U/$.
Solution. We are given: Eur. put, t = 0.5, x = 120, K = 115, rf = r$ = = 0.06, = 0.2, rd =
rU = r = 0.02.
P = Kerd t N (d2 ) xerf t N (d1 ), where

120
+ 0.02 0.06 + 0.5 0.22 0.5
ln 115
ln (x/K) + (rd rf + 12 2 )t
0.0326

d1 =
=
=
= 0.2302
0.1414
t
0.2 0.5
N (d1 ) = N (0.23) = 1 0.591 = 0.409

d2 = d1 t = 0.2302 0.1414 = 0.0888


N (d2 ) = N (0.09) = 1 0.5359 = 0.4641
Kerd t = 115e0.020.5 = 113.8557
xerf t = 120e0.060.5 = 116.4535
P = 113.8557 0.4641 116.4535 0.409 = 52.8404 47.6295 = 5.211

Problem 6
You are given:
(i) The 1-year futures price for gold is 550.
(ii) The annual volatility of the price of gold is 0.25.
(iii) The continuously compounded risk-free interest rate is 3%.
Determine the Black premium for a 1-year European call option on the futures contract with a strike
price of 580.
Page 4 of 8

Copyright Natalia A. Humphreys, 2014

ACTS 4302. AU 2014. SOLUTION TO HOMEWORK 5.

Solution. We are given: Eur. call, t = 1, F = 550, K = 580, = 0.25, r = 0.03.


C = C = F ert N (d1 ) Kert N (d2 ), where
ln (F/K) + 12 2 t
ln 550 + 0.5 0.252

= 580
= 0.0874
0.25
t
N (d1 ) = 1 N (0.09) = 1 0.5359 = 0.4641

d2 = d1 t = 0.0874 0.25 = 0.3374


N (d2 ) = 1 N (0.34) = 1 0.6331 = 0.3669

d1 =

Kert = 580e0.03 = 562.8584


F ert = 550e0.03 = 533.745
C = 533.745 0.4641 562.8584 0.3669 = 247.7111 206.5128 = 41.1983

Problem 7
You own the following portfolio of options on a stock whose price is 52:

Number of options Option price Elasticity


20

2.35

3.30

40

1.85

-2.50

50

0.70

1.90

Calculate the elasticity of this portfolio.


Solution. To calculate the value of a portfolio, take a weighted average of the elasticity of the instruments in it, rather than a sum (as you would do with option Greeks). The value of the portfolio
is:
W = 20 2.35 + 40 1.85 + 50 0.70 = 156
Each weight is
Ni Ci
Ni Ci
=
, where
W
156
Ni is the number of shares and Ci is the value of each option. Hence, the elasticity of the portfolio is:
wi =

20 2.35
40 1.85
50 0.70
2.50
+ 1.90
=
156
156
156
= 3.30 0.3013 2.50 0.4744 + 1.90 0.2244 = 0.99423 1.1859 + 0.4263 = 0.2346
= 3.30

Problem 8
For a 6-month European put option on a stock, you are given:
(i)
(ii)
(iii)
(iv)
(v)

The
The
The
The
The

stocks price is 40.


stocks continuous dividend rate is 0.03
stocks volatility is 0.4.
strike price is 44.
risk free rate is 0.06.

Determine the elasticity of the put option.


Page 5 of 8

ACTS 4302. AU 2014. SOLUTION TO HOMEWORK 5.

Copyright Natalia A. Humphreys, 2014

Solution. We are given: Eur. put, t = 0.5, S = 40, K = 44, = 0.03, = 0.4, r = 0.06.
Sput
P
= et N (d1 )

put =
put

P = Kert N (d2 ) Set N (d1 ), where



40
ln 44
+ 0.06 0.03 + 0.5 0.42 0.5
ln (S/K) + (r + 12 2 )t
0.0403

d1 =
=
=
= 0.1425
0.2828
t
0.4 0.5
N (d1 ) = N (0.14) = 0.5557
put = e0.030.5 0.5557 = 0.5473

d2 = d1 t = 0.1425 0.2828 = 0.4254


N (d2 ) = N (0.43) = 0.6664
Kert = 44e0.060.5 = 42.6996
Set = 40e0.030.5 = 39.4045
P = 42.6996 0.6664 39.4045 0.5557 = 28.455 21.8919 = 6.5631
40 0.5473
put =
= 3.3356 
6.5631
Problem 9
For a stock:
(i)
(ii)
(iii)
(iv)

The
The
The
The

price is 65.
continuous dividend rate is 0.015.
volatility of the stock is 25%.
continuously compounded risk-free rate is 3%.

A portfolio has 3 European put options on this stock.


The first allows sale of 200 shares of the stock at the end of 6 months at strike price 60.
The second allows sale of 300 shares of the stock at the end of 9 months at strike price 65.
The third allows sale of 400 shares of the stock at the end of one year at strike price 70.
Calculate the delta for this portfolio of puts.
Solution. The formula for for each put is: put = et N (d1 ). Well calculate this for each
option. It helps a little that r + 21 2 = 0.03 0.015 + 0.5 0.252 = 0.04625 is the same for all 3
options.
For the first option we have:
ln (S/K) + (r + 12 2 )t
ln 65 + 0.04625 0.5

= 60
= 0.5836
t
0.25 0.5
N (d1 ) = N (0.58) = 1 N (0.58) = 1 0.719 = 0.281

d1 =

1 = e0.0150.5 0.281 = 0.2789


For the second option we have:
ln 65
0.75
65 + 0.04625

= 0.1602
0.25 0.75
N (d1 ) = N (0.16) = 1 N (0.16) = 1 0.5636 = 0.4364
d1 =

2 = e0.0150.75 0.4364 = 0.4315


Page 6 of 8

Copyright Natalia A. Humphreys, 2014

ACTS 4302. AU 2014. SOLUTION TO HOMEWORK 5.

For the third option we have:


ln 65
70 + 0.04625
= 0.1114
0.25
N (d1 ) = N (0.11) = 0.5438
d1 =

3 = e0.015 0.5438 = 0.5357

Delta for the portfolio is the weighted sum:


portf olio = 2001 + 3002 + 4003 =
= 200 (0.2789) + 300 (0.4315) + 400 (0.5357) =
= 55.7801 129.4554 214.2815 = 399.51 

Problem 10
Five observations of the logged ratios of stock prices, in order, are:
0.06, 0.03, 0.07, 0.11, 0.17
a) Determine the y-axis label of the point on the normal probability plot corresponding to 0.03.
b) Determine the smoothed 60th percentile.
Solution. Note that since the data is given to us in ascending order, its called order statistics.
a) Let us determine the values of Fn (xi ) and corresponding values of the inverse normal cumulative
probability function (quantiles) for these observation. Since n = 5 and Fn (xi ) = (2i 1)/2n, i =
1, . . . n, Therefore, the y-axis label of the point on the normal probability plot corresponding to 0 is

xi
yi = F5 (xi )
N 1 (yi )

0.06
1
10

= 0.1

-1.28

0.03
3
10

= 0.3

-0.525

0.07
5
10

= 0.5
0

0.11
7
10

= 0.7

0.525

0.17
9
10

= 0.9

1.285

F5 (0) = 0.3.
th

b) The ith observation represents 2i1


percentile, so the third observation is the 50th percentile and
2n
the forth observation is the 70th percentile. Note that the 60th percentile should lie half way between
the third and the forth observation. Hence, the 60th percentile is:
0.07 0.5 + 0.11 0.5 = 0.09

Problem 11
You are given the following information on the price of a stock:
Page 7 of 8

Copyright Natalia A. Humphreys, 2014

ACTS 4302. AU 2014. SOLUTION TO HOMEWORK 5.

Date

Stock price

Jul. 1, 2007

35.30

Aug. 1, 2007

33.90

Sep. 1, 2007

41.20

Oct. 1, 2007

31.95

Nov. 1, 2007

38.25

Dec. 1, 2007

46.18

Estimate the annual volatility of continuously compounded return on the stock.


(A) 0.20
(B) 0.62
(C) 0.68
(D) 0.69
(E) 0.75
Key: D
Solution. To estimate volatility from historical data:
s
P
P 2




n
xt
St
xt
2

= N
x
, where xt = ln
, x
=
n1
n
St1
n
N is the number of periods per year, n is the number one less than the number of observations of stock
price.
In our problem N=12 and n=5. We calculate xt = ln(St /St1 ) and x2t :
St

xt

x2t

35.30
33.90 -0.0405 0.0016
41.20

0.1950

0.0380

31.95 -0.2543 0.0647


38.25

0.1800

0.0324

46.18

0.1884

0.0355

Summing up the third column and its squares,


5
X

X
0.2687
= 0.05373,
x2t = 0.1722
5
t=1
t=1
 


5
0.1722
s2 =
0.053732 = 0.03944, s = 0.03944 = 0.1986
4
5
That is the monthly volatility. The annual volatility is

0.1986 12 = 0.688 0.69 


xt = 0.2687, x
=

Page 8 of 8

You might also like